+ All Categories
Home > Documents > Cl i c k t o P ri nt l ogout Chal l enges...2018/05/22  · य द स झ द र व ल ख...

Cl i c k t o P ri nt l ogout Chal l enges...2018/05/22  · य द स झ द र व ल ख...

Date post: 12-Dec-2020
Category:
Upload: others
View: 3 times
Download: 0 times
Share this document with a friend
30
CANDIDATE NAME : KVCLASSES EXAM START TIME : 15:15:00 EXAM DURATION : 02:00 Hrs TOTAL NO OF QUESTIONS : 100 Click to Print logout Challenges REGISTRATION NUMBER : KVCLASSES EXAM DATE : 22-February-2018 EXAM NAME : SSC Examination TOTAL MARKS : 200 QID : 201 - If the partnership deed is silent, then what is the minimum salary or other remuneration paid to partner every month for taking part in the conduct of the business? यद साझेदार वलेख अनुचरत है , तो एक साझेदार को यवसाय के संचालन भाग लेने के लए कतना यूनतम वेतन तथा अय पारमक देना चाहए? Options: 1) Rs 10,000 10,000 2) Rs 20,000 20,000 3) 10% of capital brought by partner साझेदार वारा नवेश गई पू ँजी का 10% 4) No salary कोई वेतन नहं Correct Answer: No salary कोई वेतन नहं Candidate Answer: Rs 10,000 10,000 QID : 202 - Which of the following needs to be excluded from current assets apart from closing stock, to get the balance as liquid assets? ननलखत से कसे मौजूदा टॉक से अलग वतमान परसंपितय से बाहर रखा जाना चाहए, ताक चल परसंपित के समान राश ात जा सके ? Options: 1) Prepaid expenses पूवदत यय 2) Debtors देनदार 3) Accrued Income उपािजत आय 4) Bill receivable ाय वप Correct Answer: Prepaid expenses पूवदत यय Candidate Answer: Prepaid expenses पूवदत यय QID : 203 - Which among the following is a capital receipts? ननलखत से कौन सी एक पू ँजीगत ाित है ? Options: 1) Amount received by way of loans ऋण के ात राश 2) Money obtained from sale of goods वतुओं से ात धन 3) Dividend received on investments नवेश पर ात लाभांश 4) Commission and fees received for services rendered सेवाएँ देने से ात कमीशन तथा शुक Correct Answer: Amount received by way of loans ऋण के ात राश www.kvclasses.com Downloaded from www.kvclasses.com Visit www.kvclasses.com for tons of free study materials and latest exam updates
Transcript
  • CANDIDATE NAME : KVCLASSES EXAM START TIME : 15:15:00 EXAM DURATION : 02:00 Hrs TOTAL NO OF QUESTIONS : 100

    Click to Print logout Challenges

    REGISTRATION NUMBER : KVCLASSES EXAM DATE : 22-February-2018 EXAM NAME : SSC Examination TOTAL MARKS : 200

    QID : 201 - If the partnership deed is silent, then what is the minimum salary or other remuneration paid to partner every month for taking part in the conduct of thebusiness?

    य�द साझेदार� �वलेख अनु�च�रत है, तो एक साझेदार को �यवसाय के संचालन म� भाग लेने के �लए �कतना �यूनतम वेतन तथा अ�य पा�र��मक देना चा�हए?Options:

    1) Rs 10,00010,000 �

    2) Rs 20,00020,000 �

    3) 10% of capital brought by partner साझेदार �वारा �नवेश क� गई पूँजी का 10%

    4) No salary कोई वेतन नह�ं

    Correct Answer: No salaryकोई वेतन नह�ंCandidate Answer: Rs 10,00010,000 �

    QID : 202 - Which of the following needs to be excluded from current assets apart from closing stock, to get the balance as liquid assets?

    �न�न�ल�खत म� से �कसे मौजदूा �टॉक से अलग वत�मान प�रसंपि�तय� से बाहर रखा जाना चा�हए, ता�क चल प�रसंपि�त के समान रा�श �ा�त क� जा सके?Options:

    1) Prepaid expensesपूव�द�त �यय2) Debtorsदेनदार

    3) Accrued Incomeउपािज�त आय

    4) Bill receivable�ा�य �वप�Correct Answer: Prepaid expensesपूव�द�त �ययCandidate Answer: Prepaid expensesपूव�द�त �यय

    QID : 203 - Which among the following is a capital receipts?

    �न�न�ल�खत म� से कौन सी एक पूँजीगत �ाि�त है?Options:

    1) Amount received by way of loansऋण के �प म� �ा�त रा�श

    2) Money obtained from sale of goodsव�तुओं क� �ब�� से �ा�त धन

    3) Dividend received on investments�नवेश पर �ा�त लाभांश

    4) Commission and fees received for services renderedसेवाएँ देने से �ा�त कमीशन तथा श�ुकCorrect Answer: Amount received by way of loansऋण के �प म� �ा�त रा�श

    ww

    w.k

    vcla

    sses

    .com

    Downloaded from www.kvclasses.com

    Visit www.kvclasses.com for tons of free study materials and latest exam updates

    http://thepracticetest.in/cs_cglet2_18/logout.php?appid=1368062d74107fc05177712c90c3885chttp://164.100.78.55/challengesystem/sendrollno.jsp?rollno=3007000619&scode=401&exam_date=22-02-2018&key=8a9f08d81ae6a075b50c04ada9df1b77

  • Candidate Answer: Amount received by way of loans ऋण के �प म� �ा�त रा�श

    QID : 204 - ______ is the cost of use of things or services for the purpose of generating revenue.

    राज�व उ�प�न करने म� �योग क� गई सेवाओं अथवा व�तुओं क� लागत ______ है।Options:

    1) Assets स�प�त

    2) Profit लाभ

    3) Stock �टॉक

    4) Expense �यय

    Correct Answer: Expense �यय

    Candidate Answer: Expense �यय

    QID : 205 - ______ states that accounting report must belong to a common period and use common unit of measurement and format of reporting.

    ______ बताता है �क लेखांकन �लेख� का संबंध समान अव�ध से हो तथा वह समान मापन इकाई व समान ��तवेदन �ा�प का उपयोग कर�।Options:

    1) Relevance �ासं�गकता

    2) Comparability तुलनीयता

    3) Understand ability बोधग�यता

    4) Reliability �व�वसनीयता

    Correct Answer: Comparability तुलनीयता

    Candidate Answer: Comparability तुलनीयता

    QID : 206 - Which of the following accounting concept eliminates personal biasedness and helps in achieving results that are comparable?

    �न�न�ल�खत म� से कौन सी लेखांकन संक�पना �यि�तगत आ�ह का �नराकरण करती है तथा उन प�रणाम� को �ा�त करने म� सहायता करती है जो तुलनीय हो?Options:

    1) Accounting period लेखांकन अव�ध

    2) Dual Aspect ��वप�ीय

    3) Cost लागत

    4) Consistency सम�पता

    Correct Answer: Consistency सम�पता

    Candidate Answer: Consistency सम�पता

    ww

    w.k

    vcla

    sses

    .com

    Downloaded from www.kvclasses.com

    Visit www.kvclasses.com for tons of free study materials and latest exam updates

  • QID : 207 - Which of the following given statements are CORRECT?

    I. Increase in revenue/gain is credited. II. Increase in capital is credited.

    III. Increase in expenses is debited. IV. Increase in liabilities is credited.

    �न�न�ल�खत म� से कौन से कथन सह� ह�?

    I. राज�व/लाभ म� व�ृ� को जमा प� म� दज� �कया जाएगा।

    II. पूँजी म� व�ृ� को जमा प� म� दज� �कया जाएगा।

    III. �यय� म� व�ृ� को नाम प� म� दज� �कया जाएगा।

    IV. देयताओं म� व�ृ� को जमा प� म� दज� �कया जाएगा।Options:

    1) Only I and III केवल I तथा III

    2) Only II and IV केवल II तथा IV

    3) Only I, III and IV केवल I, III तथा IV

    4) All are correct. सभी सह� ह�।

    Correct Answer: All are correct. सभी सह� ह�।

    Candidate Answer: All are correct. सभी सह� ह�।

    QID : 208 - Returned damaged office stationary and received Rs 10,000. The account to be credited is ______.

    काया�लय क� ��त��त लेखन साम�ी को वापस लौटाया गया तथा 10,000 � �ा�त �कए गए। ______ खाते म� जमा �कया जाएगा।Options:

    1) office stationary काया�लय क� लेखन साम�ी

    2) cash रोकड़

    3) purchase return �य वापसी

    4) debtors देनदार

    Correct Answer: office stationary काया�लय क� लेखन साम�ी

    Candidate Answer: purchase return �य वापसी

    QID : 209 - Which of the following statements are FALSE?

    I. Cash receipts are entered on the credit side of cash book.

    II. Credit purchase of a land is entered in purchase journal. III. Ledger is a subsidiary book.

    IV. Petty cash book is a book having record of small payments.

    �न�न�ल�खत म� से कौन से कथन स�य नह�ं ह�?

    I. रोकड़ बह� के जमा प� म� नकद �ाि�तय� का अ�भलेखन �कया जाता है।

    II. भू�म क� उधार खर�द को �य रोजनामचे म� अ�भ�ल�खत �कया जाएगा।

    III. खाता बह� एक सहायक पु�तक है।

    IV. खुदरा रोकड़ बह� म� छोटे भुगतान� का आलेख होता है।w

    ww

    .kvc

    lass

    es.c

    om

    Downloaded from www.kvclasses.com

    Visit www.kvclasses.com for tons of free study materials and latest exam updates

  • Options: 1) Only I and II

    केवल I तथा II 2) Only II and III

    केवल II तथा III 3) Only III and IV

    केवल III तथा IV 4) Only I and IV केवल I तथा IV

    Correct Answer: Only I and II केवल I तथा II

    Candidate Answer: Only II and III केवल II तथा III

    QID : 210 - Which of the following statements is TRUE?

    �न�न�ल�खत म� से कौन सा कथन स�य है?Options:

    1) Cheques deposited but not collected will result in decreasing the balance of the cash book, when compared to passbook. चेक जमा �कए गए परंतु एक��त नह�ं �कए गए इसका प�रणाम यह होगा �क रोकड बह� म� पासबुक क� तुलना म� शषेरा�श कम हो जायेगी।

    2) Favorable bank balance as per the cash book will be less than the bank passbook balance when there are un presented cheque for payment. रोकड़ बह� के अनुसार अनुकूल ब�क शषेरा�श, ब�क पासबुक क� शषेरा�श क� तुलना कम होगी जब भुगतान के �लए चेक ��तुत नह�ं �कये जाय�गे।

    3) The credit balance of the account of the business in the books of the bank should be unequal to the debit balance of the bank account as per the cash book. �यापार के खाते म� ऋण क� शषेरा�श ब�क क� पु�तक� म� रोकड़ बह� के अनुसार ब�क खाते के देनदार शषेरा�श के बराबर नह�ं होनी चा�हए।

    4) Cheques issued but not presented for payment will reduce the balance as per the passbook. पासबुक के अनुसार जार� �कए गए चेक ले�कन भुगतान के �लए ��तुत नह�ं �कये गये, शषेरा�श को कम कर देग�।

    Correct Answer: Favorable bank balance as per the cash book will be less than the bank passbook balance when there are un presented cheque for payment. रोकड़ बह� के अनुसार अनुकूल ब�क शषेरा�श, ब�क पासबुक क� शषेरा�श क� तुलना कम होगी जब भुगतान के �लए चेक ��तुत नह�ं �कये जाय�गे।

    Candidate Answer: Cheques deposited but not collected will result in decreasing the balance of the cash book, when compared to passbook. चेक जमा �कए गए परंतु एक��त नह�ं �कए गए इसका प�रणाम यह होगा �क रोकड बह� म� पासबुक क� तुलना म� शषेरा�श कम हो जायेगी।

    QID : 211 - What is/are the objective(s) of preparing the trial balance?

    तलपट को तैयार करने का/के �या उ�े�य है/ह�?Options:

    1) To ascertain the arithmetical accuracy of the ledger accounts. बह� खात� क� अकं�य श�ुता का �नधा�रण करना।

    2) To help in locating errors. अश�ु�य� का पता लगाने म� सहायता करना।

    3) To help in the preparation of the financial statement �व�तीय �ववरण� को तैयार करने म� सहायता करना।

    4) All of these ये सभी

    Correct Answer: All of these ये सभी

    Candidate Answer: All of these ये सभी

    QID : 212 - Which among the following will be shown as the debit balance in a trial balance?

    �न�न�ल�खत म� से कौन सा तलपट म� नाम रा�श क� तरह दशा�या जाएगा?Options:

    1) Interest paid �याज का भुगतान 2) Long term loan

    द�घ�काल�न ऋण 3) Advances from customers

    �ाहक� से अ��म रा�श 4) Purchase return �य वापसी

    Correct Answer: Interest paid �याज का भुगतान

    ww

    w.k

    vcla

    sses

    .com

    Downloaded from www.kvclasses.com

    Visit www.kvclasses.com for tons of free study materials and latest exam updates

  • Candidate Answer: Interest paid �याज का भुगतान

    QID : 213 - If a credit purchase of machinery is recorded in purchase book instead of journal proper it is an error of ______.

    य�द �कसी मशीन का उधार �य �कया गया है तथा उसका अ�भलेखन मूल रोजनामचा के �थान पर �य बह� म� कर �दया गया है, तो यह ______ अश�ु� है।Options:

    1) Compensating ��तपूरक

    2) Principle सै�ां�तक

    3) Omission लोप

    4) Commission लेख

    Correct Answer: Principle सै�ां�तक

    Candidate Answer: Principle सै�ां�तक

    QID : 214 - Which of the following error, will be rectified through suspense account?

    �न�न�ल�खत म� से �कस अश�ु� का संशोधन ‘उचतं खाते’ क� सहायता से होगा?Options:

    1) Record of sales return by Lalit of Rs 5,000 in purchase return. ल�लत �वारा 5,000 � क� �व�य वापसी का अ�भलेखन �य वापसी बह� म� करना।

    2) Omission of recording sales return by Lalit of Rs 5,000. ल�लत �वारा 5,000 � क� �व�य वापसी का अ�भलेखन न करना।

    3) Sales return of Rs 5,000 by Lalit recorded as Rs 500. ल�लत �वारा 5,000 � क� �व�य वापसी को 500 � से दज� करना। 4) Underrating of sales book by Rs 5,000.

    �व�य बह� का योग 5,000 � से कम करना।Correct Answer: Underrating of sales book by Rs 5,000.

    �व�य बह� का योग 5,000 � से कम करना।Candidate Answer: Underrating of sales book by Rs 5,000.

    �व�य बह� का योग 5,000 � से कम करना। QID : 215 - Which of the following statement is FALSE in relation to the term depreciation?

    �न�न�ल�खत म� से कौन सा कथन मू�य�ास के संबंध म� अस�य है?Options:

    1) It is decline in the book value of fixed assets. यह �थायी प�रसंपि�तय� के पु�तक-मू�य का अव�यण है।

    2) It is a continuing process. यह एक �नरंतर चलने वाल� ���या है।

    3) It is an unexpired cost. यह असमा�त लागत है।

    4) It is a non cash expense. यह गरै रोकड़ �यय है।

    Correct Answer: It is an unexpired cost. यह असमा�त लागत है।

    Candidate Answer: It is a non cash expense. यह गरै रोकड़ �यय है।

    QID : 216 - ______ refers to writing off the cost of intangible assets like patents, copyrights, trademarks, franchises, and goodwill which have utility for a specifiedperiod of time.

    ______ से अ�भ�ाय पेट�ट, कापीराइट, �ेडमाक� , �� चाइज़ी, तथा �या�त, िजनका एक �नि�चत अव�ध के �लए ह� उपयोग �कया जाता है, मू�य को पु�तक� म� �यय �दखा कर समा�त करने से है।

    ww

    w.k

    vcla

    sses

    .com

    Downloaded from www.kvclasses.com

    Visit www.kvclasses.com for tons of free study materials and latest exam updates

  • Options: 1) Depreciation

    �ास 2) Diminution

    कमी 3) Depletion

    �रि�तकरण 4) Amortisation

    प�रशोधनCorrect Answer: Amortisation

    प�रशोधनCandidate Answer: Amortisation

    प�रशोधन QID : 217 - Which of the following statement is FALSE?

    �न�न�ल�खत म� से कौन सा कथन अस�य है?Options:

    1) Extraction of natural resources like mines, quarries etc. that reduces the availability of the quantity of the material or asset is termed as depletion. �रि�तकरण श�द का �योग �ाकृ�तक साधन� को खोदकर �नकालने के संदभ� म� �कया जाता है जसेै खान�, खदान� इ�या�द िजससे माल अथवा प�रसंपि�त क� मा�ा क� उपल�धता घट जाती है।

    2) Depreciation doesn’t provide fund for replacement. �ास पुन�था�पन के �लए कोष नह�ं जटुाता है।

    3) Depreciation need not be charged if adequate maintenance expenditure is also incurred. मू�य�ास लगाने क� आव�यकता तब नह�ं होती है य�द रख-रखाव पर ठ�क ठ�क �यय �कया जाता है।

    4) Depreciation is charged even if market value of an asset is higher than its book value. जब प�रसंपि�त का बाजार मू�य इसके पु�तक�य मू�य से अ�धक हो तो इस पर �ास लगाया जाता है।

    Correct Answer: Depreciation doesn’t provide fund for replacement. �ास पुन�था�पन के �लए कोष नह�ं जटुाता है। [OR] Depreciation need not be charged if adequate maintenance expenditure is also incurred.

    मू�य�ास लगाने क� आव�यकता तब नह�ं होती है य�द रख-रखाव पर ठ�क ठ�क �यय �कया जाता है।Candidate Answer: Depreciation need not be charged if adequate maintenance expenditure is also incurred.

    मू�य�ास लगाने क� आव�यकता तब नह�ं होती है य�द रख-रखाव पर ठ�क ठ�क �यय �कया जाता है। QID : 218 - Reserves and surpluses are shown on which side and under which head in the balance sheet?

    संचय एवं आ�ध�य, तुलन-प� म� �कस प� क� ओर तथा �कस शीष�क के अतंग�त दशा�ये जाते ह�?Options:

    1) Liability side and under capital head. देयता प� तथा पूँजी क� मद के प�चात

    2) Asset side and under investment head संपि�त प� तथा �नवेश क� मद के प�चात

    3) Liability side and under provisions. देयता प� तथा �ावधान के प�चात

    4) Asset side and under cash in hand संपि�त प� तथा रोकड़ शषे के प�चात

    Correct Answer: Liability side and under capital head. देयता प� तथा पूँजी क� मद के प�चात

    Candidate Answer: Asset side and under investment head संपि�त प� तथा �नवेश क� मद के प�चात

    QID : 219 - Which of the following statement is NOT TRUE in regard to reserve and provision?

    �न�न�ल�खत म� से कौन सा कथन संचय एवं �ावधान के संदभ� म� स�य नह�ं है?Options:

    1) A provision is a charge against profit whereas reserve is an appropriation of profit. �ावधान लाभ पर लगाया जाता है जब�क संचय लाभ का समायोजन है।

    2) Provision reduces taxable profit whereas reserves increase taxable profit. �ावधान से कर यो�य लाभ कम हो जाता है जब�क संचय से कर यो�य लाभ बढ़ जाता है। 3) Provision cannot be used for distribution as dividend whereas reserve can be used for dividend distribution.

    लाभांश के भुगतान के �लए �ावधन को लाभांश के �प म� नह�ं बाँटा जा सकता जब�क संचय को लाभांश के �लए �योग �कया जा सकता है। 4) Provision is made for a known liability or expense pertaining to current accounting period where as reserve is created for strengthening the financial position of

    business. चालू लेखांकन अव�ध से संबं�धत देनदा�रय� या खच� के �लए �ावधान बनाया जाता है जब�क संचय का सजृन �यवसाय क� �व�तीय ि�थ�त को सु�ढ़ करने के �लए �कया जाता है।

    Correct Answer: Provision reduces taxable profit whereas reserves increase taxable profit. �ावधान से कर यो�य लाभ कम हो जाता है जब�क संचय से कर यो�य लाभ बढ़ जाता है।

    ww

    w.k

    vcla

    sses

    .com

    Downloaded from www.kvclasses.com

    Visit www.kvclasses.com for tons of free study materials and latest exam updates

  • Candidate Answer: Provision reduces taxable profit whereas reserves increase taxable profit. �ावधान से कर यो�य लाभ कम हो जाता है जब�क संचय से कर यो�य लाभ बढ़ जाता है।

    QID : 220 - Provision is ______ against profit.

    �ावधान लाभ पर ______ है।Options:

    1) disposal �नराकरण

    2) residual अव�श�ट

    3) appropriation �व�नयोग

    4) charge �भार

    Correct Answer: charge �भार

    Candidate Answer: charge �भार

    QID : 221 - What will the entery for cash sales of Rs 2500?

    2500 � के नकद �व�य क� ��वि�ट �या होगी?Options:

    1) Cash A/c Dr. - Rs 2,500 To Sales A/c - Rs 2,500

    रोकड़ खाता नाम - 2,500 �

    �व�य खाते से - 2,500 �

    2) Owner's A/c Dr. - Rs 2,500 To Sales A/c - Rs 2,500

    मा�लक खाता नाम - 2,500 �

    �व�य खाते से - 2,500 �

    3) Sales A/c Dr. - Rs 2,500 To Cash A/c - Rs 2,500

    �व�य खाता नाम - 2,500 �

    रोकड़ खाते से - 2,500 �

    4) Purchases A/c Dr. - Rs 2,500 To Sales A/c - Rs 2,500

    �य खाता नाम - 2,500 �

    �व�य खाते से - 2,500 �

    Correct Answer: Cash A/c Dr. - Rs 2,500 To Sales A/c - Rs 2,500

    रोकड़ खाता नाम - 2,500 �

    �व�य खाते से - 2,500 �

    Candidate Answer: Sales A/c Dr. - Rs 2,500 To Cash A/c - Rs 2,500

    �व�य खाता नाम - 2,500 �

    रोकड़ खाते से - 2,500 �

    QID : 222 - Which account will be credited in the books of drawer when the drawer gets the bill discounted from the bank (At the time of accepting the bill)?

    आहता� �वारा ब�क से �वप� भुनाने पर आहता� क� पु�तक म� �कस खाते को जमा �कया जायेगा (�वप� क� �वीकृ�त के समय)?

    ww

    w.k

    vcla

    sses

    .com

    Downloaded from www.kvclasses.com

    Visit www.kvclasses.com for tons of free study materials and latest exam updates

  • Options: 1) Bill Receivable Account

    �ा�य �वप� खाता 2) Bank Account ब�क खाता

    3) Debtors Account देनदार खाता

    4) Discount Account छूट खाता

    Correct Answer: Debtors Account देनदार खाता

    Candidate Answer: Bill Receivable Account �ा�य �वप� खाता

    QID : 223 - When the bill is endorsed by the drawer in favour of his creditor, then which entry will be passed in the books of drawer on the maturity of the bill?

    जब �बल आदेशक �वारा उसके लेनदार के प� म� सम�थ�त �कया जाता है, तब �बल क� प�रप�वता होने पर �कस ��वि�ट को आदेशक क� पु�तक म� पा�रत �कया जाएगा?Options:

    1) Creditors Account Dr. To bill Receivable Account लेनदार खाता नाम �ा�य �वप� खाते से

    2) Bill Receivable Account Dr. to debtor’s Account �ा�य �वप� खाता नाम देनदार खाते से

    3) Debtor’s Account Dr. to sales Account देनदार खाता नाम �व�य खाते से

    4) No entry कोई ��वि�ट नह�ं

    Correct Answer: No entry कोई ��वि�ट नह�ं

    Candidate Answer: Bill Receivable Account Dr. to debtor’s Account �ा�य �वप� खाता नाम देनदार खाते से

    QID : 224 - Which of the following statement is INCORRECT about capital expenditure?

    �न�न�ल�खत म� से कौन सा कथन पूँजी �यय के संदभ� म� सह� नह�ं है?Options:

    1) It maintains the earning capacity of business. यह �यापार क� आय �मता को यथाि�थ�त म� बनाये रखता है।

    2) It is recorded in balance sheet. यह तुलन-प� पर �लखा जाता है।

    3) It is a non recurring expenditure. यह अनावत� �यय है।

    4) It gives benefit for more than one accounting year यह एक से अ�धक लेखांकन स� के �लए लाभकार� होता है।

    Correct Answer: It maintains the earning capacity of business. यह �यापार क� आय �मता को यथाि�थ�त म� बनाये रखता है।

    Candidate Answer: It is a non recurring expenditure. यह अनावत� �यय है।

    QID : 225 - According to Partnership Act 1932, which account is debited for the transfer of net loss?

    साझेदार� अ�ध�नयम 1932 के अनुसार, �नवल (नेट) हा�न के ह�तांतरण को �कस खाते के नाम म� दज� �कया जाता है?Options:

    1) Profit and loss account लाभ तथा हा�न खाता

    2) Capital account पूँजी खाता

    3) Cash account रोकड़ खाता

    4) Owner’s account �वामी खाता

    Correct Answer: Capital account पूँजी खाता

    ww

    w.k

    vcla

    sses

    .com

    Downloaded from www.kvclasses.com

    Visit www.kvclasses.com for tons of free study materials and latest exam updates

  • Candidate Answer: Owner’s account �वामी खाता

    QID : 226 -

    Options:

    1) I-2, II-4, III-3, IV-1

    2) I-3, II-4, III-2, IV-1

    3) I-2, II-1, III-3, IV-4

    4) I-3, II-1, III-4, IV-2 Correct Answer: I-3, II-1, III-4, IV-2

    Candidate Answer: I-3, II-1, III-4, IV-2

    QID : 227 - Which account is debited while recording an outstanding expense?

    बकाया �यय के अ�भलेखन के दौरान �कस खाते को नाम प� म� दज� �कया जाता है?Options:

    1) Concerned expense account संबं�धत �यय खाता

    2) Cash account रोकड़ खाता

    3) Outstanding expenses account बकाया �यय खाता

    4) Capital account पूँजी खाता

    Correct Answer: Concerned expense account संबं�धत �यय खाता

    Candidate Answer: Concerned expense account संबं�धत �यय खाता

    QID : 228 - If the insurance premium paid was Rs 2,000 and pre-paid insurance already is Rs 500, then the amount of insurance premium shown in profit lossaccount will be:

    य�द बीमा �ी�मयम का 2,000 � भुगतान �कया गया है तथा पूव�द�त बीमा 500 � है, तो लाभ व हा�न खाते म� दशा�यी गई बीमा �ी�मयम क� रा�श होगीः

    ww

    w.k

    vcla

    sses

    .com

    Downloaded from www.kvclasses.com

    Visit www.kvclasses.com for tons of free study materials and latest exam updates

  • Options: 1) Rs 2000

    2000 � 2) Rs 500

    500 � 3) Rs 1500

    1500 � 4) Rs 2500

    2500 �Correct Answer: Rs 1500

    1500 �Candidate Answer: Rs 2500

    2500 � QID : 229 - Which ratios are calculated for measuring the efficiency of operation of business based on effective utilisation of resources?

    कौन सा अनुपात संसाधन� के �भावी उपयो�गता पर आधा�रत �यवसाय क� स��यता या काया��मकता क� �मता के मापन हेतु प�रक�लत �कया जाता है?Options:

    1) Liquidity ratios �वता अनुपात

    2) Solvency ratios स�प�नता अनुपात

    3) Activity ratios स��यता अनुपात

    4) Profitability ratios लाभ�दता अनुपात

    Correct Answer: Activity ratios स��यता अनुपात

    Candidate Answer: Activity ratios स��यता अनुपात

    QID : 230 - Which of the following ratio is also termed as leverage ratio?

    �न�न�ल�खत म� से �कस अनुपात को उ�तोलन अनुपात भी कहा जाता है?Options:

    1) Debt equity ratio ऋण समता अनुपात

    2) Solvency ratio स�प�नता अनुपात 3) Proprietary ratio

    �वा�म�व अनुपात 4) Interest coverage ratio

    �याज �याि�त अनुपातCorrect Answer: Debt equity ratio

    ऋण समता अनुपातCandidate Answer: Debt equity ratio

    ऋण समता अनुपात QID : 231 - Proceeds from sale of patent is classified under which activity in Cash Flow Statement?

    पेट�ट क� �ब�� से �ाि�तय� को रोकड़ �वाह �ववरण म� �कस ��याकलाप के अतंग�त वग�कृत �कया जाता है?Options:

    1) Investing �नवेश

    2) Operating �चालन

    3) Financing �व�तीय

    4) Cash रोकड़

    Correct Answer: Investing �नवेश

    ww

    w.k

    vcla

    sses

    .com

    Downloaded from www.kvclasses.com

    Visit www.kvclasses.com for tons of free study materials and latest exam updates

  • Candidate Answer: Financing �व�तीय

    QID : 232 - Among the following items given which is/are needed to be added to get net cash from operating activities while preparing Cash Flow Statement?

    I. Increase in current liabilities

    II. Decrease in current liabilities III. Increase in current assets

    IV. Income tax paid

    �न�न�ल�खत म� से कौन सी मद/मद� को रोकड़ �वाह �ववरण तैयार करने के दौरान �चालन ��याकलाप �वारा �नवल (नेट) रोकड़ �ा�त करने के �लए जोड़ा जाता है?

    I. चालू देनदा�रय� म� व�ृ�

    II. चालू देनदा�रय� म� कमी

    III. चालू प�रसंपि�तय� म� व�ृ�

    IV. आयकर भुगतानOptions:

    1) Only I केवल I

    2) Only I and III केवल I तथा III

    3) Only II and IV केवल II तथा IV

    4) Only III केवल III

    Correct Answer: Only I केवल I

    Candidate Answer: Only I and III केवल I तथा III

    QID : 233 - In order to compensate the investors, what kind of debentures are issued at substantial discount and the difference between the nominal value and theissue price is treated as the amount of interest related to the duration of the debentures?

    �नवेशक� क� ��तपू�त � करने क� �ि�ट से �कस �कार के ऋणप� पया��त ब�े (छूट) के साथ जार� �कए जाते ह� और सांके�तक मू�य तथा �नग�म मू�य के म�य अतंर को �याज क� रा�श के �प म��न��पत �कया जाता है, जो ऋणप� क� कालाव�ध से संबं�धत होती है?Options:

    1) Zero coupon rate debenture श�ूय कूपन दर ऋणप�

    2) Bearer debentures धारक ऋणप�

    3) Specific coupon rate debentures �व�श�ट कूपन दर ऋणप�

    4) Naked debentures साधारण ऋणप�

    Correct Answer: Zero coupon rate debenture श�ूय कूपन दर ऋणप�

    Candidate Answer: Specific coupon rate debentures �व�श�ट कूपन दर ऋणप�

    QID : 234 - If the value of debentures is less than the value of the net asset taken over, then the difference will be credited to:

    य�द ऋणप� का मू�य ल� गई �नवल (नेट) प�रसंपि�त के मू�य से कम होता है, तो वहाँ इस अतंर को जमा �कया जाएगाःOptions:

    1) Goodwill account �या�त खाते म�

    2) Capital reserve account पूँजी आर��त (संचय) खाते म� 3) General reserve account सामा�य संचय खाते म�

    4) Debenture account ऋणप� खाते म�

    Correct Answer: Capital reserve account पूँजी आर��त (संचय) खाते म�

    ww

    w.k

    vcla

    sses

    .com

    Downloaded from www.kvclasses.com

    Visit www.kvclasses.com for tons of free study materials and latest exam updates

  • Candidate Answer: Goodwill account �या�त खाते म�

    QID : 235 - The part of capital which is called-up only on winding up is called ______.

    पूँजी का वह भाग जो�क समापन के समय माँगा जाता है, ______ कहलाता है।Options:

    1) Reserve capital आर��त पूँजी

    2) General reserve सामा�य संचय

    3) Capital reserve पूँजी संचय

    4) Capital redemption reserve पूँजी शोधन संचय

    Correct Answer: Reserve capital आर��त पूँजी

    Candidate Answer: Capital redemption reserve पूँजी शोधन संचय

    QID : 236 - From which of the following, companies cannot buy its own shares?

    �न�न�ल�खत म� से �कसम�, कंपनी अपने �वयं के अशं (शयेर) नह�ं खर�द सकती?Options:

    1) Odd lot shareholders �यून खेप अशंधारक� से

    2) Open market खुले बाज़ार से

    3) Existing equity shareholders on a disproportionate basis असंगता के आधार पर वत�मान समता अशंधारक� से

    4) Employees of the company कंपनी के कम�चा�रय� से

    Correct Answer: Existing equity shareholders on a disproportionate basis असंगता के आधार पर वत�मान समता अशंधारक� से

    Candidate Answer: Employees of the company कंपनी के कम�चा�रय� से

    QID : 237 - A and B share profit in the ratio of 5 : 2. C is admitted as a partner who sets 1/7 share. If C acquires 3/28 from A and 1/28 from B, then what will be thenew profit sharing ratio?

    A तथा B लाभ का �वभाजन 5 : 2 के अनुपात म� करते है। C एक साझेदार के तौर पर 1/7 �ह�से के साथ �वेश लेता है। य�द C 3/28 A से तथा 1/28 B से लेता है, तो नया लाभ �वभाजनअनुपात �या होगा?Options:

    1) 19 : 5 : 4 2) 17 : 7 : 4 3) 15 : 9 : 4 4) 16 : 8 : 4

    Correct Answer: 17 : 7 : 4Candidate Answer: 17 : 7 : 4 QID : 238 - Sales revenue is a:

    �ब�� राज�व हैःOptions:

    1) Real account वा�त�वक खाता

    2) Nominal account अवा�त�वक खाता

    3) Personal account �यि�तगत खाता

    4) Both Real and Nominal account वा�त�वक तथा अवा�त�वक खाता दोन�

    Correct Answer: Real account वा�त�वक खाता

    ww

    w.k

    vcla

    sses

    .com

    Downloaded from www.kvclasses.com

    Visit www.kvclasses.com for tons of free study materials and latest exam updates

  • Candidate Answer: Nominal account अवा�त�वक खाता

    QID : 239 - Which of the following formulae is INCORRECT?

    �न�न�ल�खत म� से कौन सा सू� सह� नह�ं है?Options:

    1) Gain in share of continuing partner = New share –old share �व�यामान साझेदार� के �ह�से म� बढ़ो�तर� = नया �ह�सा – पुराना �ह�सा 2) Average collection period = 365/Creditors turnover ratio

    औसत सं�हण अव�ध = 365/ऋणदाताओं के कारोबार का अनुपात 3) Operating expenses = Cost of goods sold + selling expenses + Administrative expenses

    �चालन �यय = बेची गई व�तुओं क� लागत + �व�य �यय + �शास�नक �यय 4) Number of shares to be issued = Amount Payable / Issue Price

    �नग�मन �कए जाने वाले अशं� (शयेर�) क� सं�या = देय रा�श/�नग�मन मू�यCorrect Answer: Average collection period = 365/Creditors turnover ratio

    औसत सं�हण अव�ध = 365/ऋणदाताओं के कारोबार का अनुपातCandidate Answer: Operating expenses = Cost of goods sold + selling expenses + Administrative expenses

    �चालन �यय = बेची गई व�तुओं क� लागत + �व�य �यय + �शास�नक �यय QID : 240 - Interest Coverage Ratio and proprietary ratio comes under:

    �याज �याि�त अनुपात तथा �वा�म�व अनुपात �न�न�ल�खत के अतंग�त आता हैःOptions:

    1) solvency ratio स�प�नता अनुपात 2) activity ratio

    स��यता अनुपात 3) profitability ratio

    लाभ�दता अनुपात 4) liquidity ratio �वता अनुपात

    Correct Answer: solvency ratio स�प�नता अनुपात

    Candidate Answer: liquidity ratio �वता अनुपात

    QID : 241 - Who is the current CAG of India?

    भारत के वत�मान सीएजी कौन है?Options:

    1) Vinod Rai �वनोद राय

    2) V. N. Kaul वी. एन. कौल

    3) Shashikant Sharma श�शकांत शमा�

    4) Rajiv Mehrishi राजीव मह�ष�

    Correct Answer: Rajiv Mehrishi राजीव मह�ष�

    Candidate Answer: Shashikant Sharma श�शकांत शमा�

    QID : 242 - Which part of Indian constitution establishes CAG as an authority?

    भारतीय सं�वधान का कौन सा भाग सीएजी को एक �ा�धकार� घो�षत करता है?Options:

    1) V

    2) VIII

    3) XV

    4) XX

    ww

    w.k

    vcla

    sses

    .com

    Downloaded from www.kvclasses.com

    Visit www.kvclasses.com for tons of free study materials and latest exam updates

  • Correct Answer: V Candidate Answer: V

    QID : 243 - The CAG once appointed has to make an oath or affirmation before ________.

    सी.ए.जी. (कैग) क� �नयुि�त के प�चात वह _______ के सम� शपथ �हण या ��त�ा करते है।Options:

    1) The Prime Minister of India भारत के �धानमं�ी

    2) The President of India भारत के रा��प�त

    3) The Supreme court Judge उ�चतम �यायालय के �यायाधीश 4) The Finance Minister of India

    भारत के �व�त मं�ीCorrect Answer: The President of India

    भारत के रा��प�तCandidate Answer: The President of India

    भारत के रा��प�त QID : 244 - Who determines the term of office, salary and allowances, conditions of service of the chairman and other members of the Finance Commission of Indiafrom time to time?

    भारत के �व�त आयोग के अ�य� तथा अ�य सद�य� के काय�काल, वेतन एवं भ�ते तथा सेवा क� शत� का समय समय पर �नधा�रण कौन करता है?Options:

    1) The President of India भारत के रा��प�त

    2) The Prime Minister of India भारत के �धानमं�ी

    3) The Finance Minister of India भारत के �व�त मं�ी

    4) Governor of RBI आरबीआई के गवन�र

    Correct Answer: The President of India भारत के रा��प�त

    Candidate Answer: The President of India भारत के रा��प�त

    QID : 245 - What is the requirement of other members of the finance commission fixed under the Article 280 of the Constitution of India apart from the its chairman?

    भारत के सं�वधान के अनु�छेद 280 के तहत �नधा��रत �व�त आयोग के अ�य� के अलावा अ�य सद�य� क� आव�यकता �कतनी है?Options:

    1) Four चार

    2) Five पाँच

    3) Six छह

    4) Eight आठ

    Correct Answer: Four चार

    Candidate Answer: Four चार

    QID : 246 - Which among the following is one of the three heads grouped as recommendations of the finance commission.

    �न�न�ल�खत म� से कौन �व�त आयोग क� �सफा�रश� के अनुसार वग�कृत तीन �मुख� म� से एक है?

    ww

    w.k

    vcla

    sses

    .com

    Downloaded from www.kvclasses.com

    Visit www.kvclasses.com for tons of free study materials and latest exam updates

  • Options: 1) Union profits to states

    रा�य� को संघीय लाभ 2) Central Loan to states

    रा�य� को के���य ऋण 3) Central Tax to states

    रा�य� को के���य कर 4) Union Loan to states

    रा�य� को संघीय ऋणCorrect Answer: Union Loan to states

    रा�य� को संघीय ऋणCandidate Answer: Central Loan to states

    रा�य� को के���य ऋण QID : 247 - Decrease of resources implies that production possibility curve:

    संसाधन� क� कमी का अथ� है �क उ�पादन के संभावना व� का:Options:

    1) shifts to right दायीं ओर �खसकना 2) shifts to left

    बायीं ओर �खसकना 3) rotates to the right

    दायीं ओर घूमना 4) rotates to the left

    बायीं ओर घूमनाCorrect Answer: shifts to left

    बायीं ओर �खसकनाCandidate Answer: shifts to left

    बायीं ओर �खसकना QID : 248 - Which of the following relation is INCORRECT between TU and MU?

    �न�न�ल�खत म� से कौन सा संबंध ट�यू तथा एमयू के म�य सह� नह�ं है?Options:

    1) TU increase as long as MU is positive. ट�यू म� व�ृ� होती है जब तक एमयू सकारा�मक होता है।

    2) TU is maximum when MU = 0. ट�यू अ�धकतम होगा जब एमयू = 0 हो। 3) TU starts declining when MU is negative.

    एमयू नकारा�मक होने पर ट�यू कम होना आरंभ हो जाता है। 4) Decrease in MU implies that TU increases at an increasing rate.

    एमयू म� कमी का अथ� है �क ट�यू बढ़ती दर से बढ़ता है।Correct Answer: Decrease in MU implies that TU increases at an increasing rate.

    एमयू म� कमी का अथ� है �क ट�यू बढ़ती दर से बढ़ता है।Candidate Answer: Decrease in MU implies that TU increases at an increasing rate.

    एमयू म� कमी का अथ� है �क ट�यू बढ़ती दर से बढ़ता है। QID : 249 - Indifference Curve is ______ at the point of equilibrium.

    उदासीनता व� संतुलन के �ब�द ुपर ______ होता है।Options:

    1) convex उ�तल

    2) concave अवतल

    3) straight सीधा

    4) zero श�ूय

    Correct Answer: convex उ�तल

    ww

    w.k

    vcla

    sses

    .com

    Downloaded from www.kvclasses.com

    Visit www.kvclasses.com for tons of free study materials and latest exam updates

  • Candidate Answer: convex उ�तल

    QID : 250 - The two aspects: (1) Factoral distribution of Income. (2) Inter-personal distribution of Income are parts of which of the following central problem of aneconomy?

    दो पहलुओः (1) आय का �मगु�णत �वतरण (2) आय का अतंर-�यि�तगत �वतरण अथ��यव�था क� �न�न�ल�खत म� से कौन सी क� ��य सम�या का �ह�सा है?Options:

    1) What to produce? �या उ�पादन कर�?

    2) How to produce? कैसे उ�पादन कर�?

    3) For whom to produce? �कसके �लए उ�पादन कर�?

    4) Why to produce? �य� उ�पादन कर�?

    Correct Answer: For whom to produce? �कसके �लए उ�पादन कर�?

    Candidate Answer: What to produce? �या उ�पादन कर�?

    QID : 251 - In which type of economy social justice is accorded higher priority than profit maximization?

    �कस �कार क� अथ��यव�था म� सामािजक �याय को लाभ क� अ�धकतमता से उ�च �ाथ�मकता द� जाती है?Options:

    1) Centrally Planned Economy के���य �प से �नयोिजत अथ��यव�था

    2) Mixed Economy �म��त अथ��यव�था 3) Capitalist Economy

    पूँजीवाद� अथ��यव�था 4) Private Economy �नजी अथ��यव�था

    Correct Answer: Centrally Planned Economy के���य �प से �नयोिजत अथ��यव�था

    Candidate Answer: Centrally Planned Economy के���य �प से �नयोिजत अथ��यव�था

    QID : 252 - A situation when demand curve shifts to the right is described under the following situation:

    एक ि�थ�त जब माँग व� दायीं ओर �खसकता है तो उसे �न�न�ल�खत ि�थ�त म� व�ण�त �कया जाता हैःOptions:

    1) extension in demand माँग म� �व�तार

    2) increase in demand माँग म� व�ृ�

    3) contraction in demand माँग म� संकुचन

    4) decrease in demand माँग म� कमी

    Correct Answer: increase in demand माँग म� व�ृ�

    Candidate Answer: extension in demand माँग म� �व�तार

    QID : 253 - What is the total elasticity of demand, if a fall in own price of a commodity causes a fall in total expenditure and a rise in price causes rise in totalexpenditure of the commodity?

    माँग क� कुल लोच �या है, य�द �कसी व�तु के अपने मू�य म� �गरावट होने से कुल �यय म� �गरावट हो जाती है तथा मू�य म� व�ृ� होने से व�तु के कुल �यय म� व�ृ� हो जाती है?

    ww

    w.k

    vcla

    sses

    .com

    Downloaded from www.kvclasses.com

    Visit www.kvclasses.com for tons of free study materials and latest exam updates

  • Options: 1) Less than unitary

    एका�मक से कम 2) Greater than unitary

    एका�मक से अ�धक 3) Unitary

    एका�मक 4) Perfectly inelastic

    पूण�तः बेलोचCorrect Answer: Less than unitary

    एका�मक से कमCandidate Answer: Unitary

    एका�मक QID : 254 - Suppose the demand function for a commodity is given as Q= 1000 – 10P where ‘Q’ denotes quantity of demand and ‘P’ denotes price of the commodity.The Point Price elasticity of demand at price Rs 20 will be:

    मान ल�िजए �क �कसी व�तु के �लए माँग काय� Q = 1000 – 10P के �प म� �दया गया है जहाँ ‘Q’ माँग क� मा�ा को दशा�ता है तथा ‘P’ व�तु के मू�य को दशा�ता है। 20 � के मू�य पर माँग का�ब�द ुमू�य लोच होगाःOptions:

    1) – 0.50

    2) – 0.20

    3) – 0.25

    4) – 5 Correct Answer: – 0.25

    Candidate Answer: – 0.20

    QID : 255 - “Supply creates its own demand” is a ______.

    “आपू�त � अपनी माँग बनाती है” एक ______ है।Options:

    1) Pareto principle पारेटो �स�ांत

    2) Martian law माश�न �नयम

    3) Say’s law सेज़ �नयम

    4) Keynes law क��स �नयम

    Correct Answer: Say’s law सेज़ �नयम

    Candidate Answer: Keynes law क��स �नयम

    QID : 256 - In the case of Veblen goods, demand curve will slope:

    वेबलेन व�तुओं क� ि�थ�त म�, माँग व� क� ढलान होगीःOptions:

    1) Upwards ऊपर क� ओर 2) Downwards

    नीचे क� ओर 3) Straight सीधी

    4) Negative नकारा�मक

    Correct Answer: Upwards ऊपर क� ओर

    Candidate Answer: Upwards ऊपर क� ओर

    ww

    w.k

    vcla

    sses

    .com

    Downloaded from www.kvclasses.com

    Visit www.kvclasses.com for tons of free study materials and latest exam updates

  • QID : 257 - Increase in the price of a competing product will lead to:

    एक ��त�पध� उ�पाद के मू�य म� व�ृ� करने पर होगीःOptions:

    1) contraction of supply आपू�त � का संकुचन

    2) increase in supply आपू�त � म� व�ृ�

    3) decrease in supply आपू�त � म� कमी

    4) extension of supply आपू�त � का �व�तार

    Correct Answer: decrease in supply आपू�त � म� कमी

    Candidate Answer: increase in supply आपू�त � म� व�ृ�

    QID : 258 - Es< 1, when positively sloped supply curve starts from ______.

    Es< 1 होता है, जब सकारा�मक झुका हुआ आपू�त � व� ______ से आरंभ होता है।Options:

    1) x-axis x-अ�

    2) y-axis y-अ�

    3) the point of origin मूल �ब�द ुसे

    4) None of these इनम� से कोई नह�ं

    Correct Answer: x-axis x-अ�

    Candidate Answer: the point of origin मूल �ब�द ुसे

    QID : 259 - Price elasticity of supply of a good is 2. By what percentage should its price rise so that its supply rises by 40 percent?

    एक व�तु क� आपू�त � क� क�मत लोच 2 है। इसक� क�मत म� �कतने ��तशत से व�ृ� होनी चा�हए ता�क इसक� आपू�त � 40 ��तशत से बढ़ जाए?Options:

    1) 20

    2) 10

    3) 50

    4) 80 Correct Answer: 20

    Candidate Answer: 20

    QID : 260 - What will be the effect on equilibrium price if supply is increased without any change in demand?

    य�द माँग म� कोई प�रवत�न �कए �बना आपू�त � बढ़ जाती है तो संतुलन क�मत पर �या �भाव पड़गेा?Options:

    1) Price will fall क�मत �गर जाएगी 2) Price will rise क�मत बढ़ेगी

    3) No change in price क�मत म� कोई प�रवत�न नह�ं होगा

    4) None of these इनम� से कोई नह�ं

    Correct Answer: Price will fall क�मत �गर जाएगी

    Candidate Answer: Price will fall क�मत �गर जाएगी

    ww

    w.k

    vcla

    sses

    .com

    Downloaded from www.kvclasses.com

    Visit www.kvclasses.com for tons of free study materials and latest exam updates

  • QID : 261 - ______ refers to different possible quantities of a commodity that the consumer is ready to buy at different possible prices of that commodity.

    ______ एक व�तु क� �व�भ�न संभव मा�ा को संद�भ�त करता है जो उपभो�ता उस व�तु के �व�भ�न संभव मू�य� पर खर�दने के �लए तैयार है।Options:

    1) desire इ�छा

    2) demand माँग

    3) quantity demanded माँगी गई मा�ा

    4) None of these इनम� से कोई नह�ं

    Correct Answer: demand माँग

    Candidate Answer: demand माँग

    QID : 262 - In case of law of variable proportions, point of inflexion is where:

    चर अनुपात के �नयम के मामले म�, मुड़ने क� �ब�द ुवहाँ है, जहां� -Options:

    1) TP starts declining because MP is negative. ट�पी कम होना आरंभ करता है �य��क एमपी नकारा�मक है।

    2) TP stops increasing at the increasing rate and starts increasing at the decreasing rate. ट�पी बढ़ती दर पर बढ़ना रोक देता है तथा घटती दर पर बढ़ना आरंभ करता है।

    3) MP is decreasing and TP is increasing at the increasing rate. एमपी कम होता है तथा ट�पी बढ़ती दर पर बढ़ता है।

    4) MP is increasing and TP is increasing at the increasing rate. एमपी बढ़ता है तथा ट�पी बढ़ती दर पर बढ़ता है।

    Correct Answer: TP stops increasing at the increasing rate and starts increasing at the decreasing rate. ट�पी बढ़ती दर पर बढ़ना रोक देता है तथा घटती दर पर बढ़ना आरंभ करता है।

    Candidate Answer: TP stops increasing at the increasing rate and starts increasing at the decreasing rate. ट�पी बढ़ती दर पर बढ़ना रोक देता है तथा घटती दर पर बढ़ना आरंभ करता है।

    QID : 263 - Which among the following is/are the important component(s) of Fixed Cost?

    �न�न�ल�खत म� से कौन सा/से त�व ि�थर लागत के मह�वपूण� घटक है/ह�?Options:

    1) Wages and salaries of permanent staff. मज़दरू� तथा �थायी कम�चा�रय� के वेतन

    2) License fee and related expenses लाइस�स श�ुक तथा संबं�धत �यय

    3) Installation charges on machine and plant मशीन तथा संयं� पर �थापना श�ुक

    4) All of these ये सभी

    Correct Answer: All of these ये सभी

    Candidate Answer: All of these ये सभी

    QID : 264 - Average Fixed Cost Curve is:

    औसत ि�थर लागत व� है:Options:

    1) Straight line सीधी रेखा

    2) Concave to the origin मूल �ब�द ुसे अवतल

    3) Rectangular Hyperbola आयताकार अ�तपरवलय

    4) None of these इनम� से कोई नह�ं

    ww

    w.k

    vcla

    sses

    .com

    Downloaded from www.kvclasses.com

    Visit www.kvclasses.com for tons of free study materials and latest exam updates

  • Correct Answer: Rectangular Hyperbola आयताकार अ�तपरवलय

    Candidate Answer: Straight line सीधी रेखा

    QID : 265 - Which of the assumption is NOT TRUE in relation to isoquant curve?

    समो�पाद व� के संबंध म� कौन सी धारणा स�य नह�ं है?Options:

    1) There are only two inputs labour and capital. केवल दो आगत मज़दरू तथा पूँजी ह�।

    2) Both inputs are indivisible. दोन� आगत� अ�वभा�य ह�।

    3) Both are imperfect substitutes for each other. दोन� एक दसूरे के �लए अपूण� �वक�प ह�।

    4) Technology of production is constant. उ�पादन क� तकनीक ि�थर है

    Correct Answer: Both inputs are indivisible. दोन� आगत� अ�वभा�य ह�।

    Candidate Answer: There are only two inputs labour and capital. केवल दो आगत मज़दरू तथा पूँजी ह�।

    QID : 266 - Margin of safety is ______ the BEP.

    सुर�ा का सीमांत BEP से ______ है।Options:

    1) below नीचे

    2) above ऊपर

    3) at par with बराबर

    4) negative of नकारा�मक

    Correct Answer: above ऊपर

    Candidate Answer: at par with बराबर

    QID : 267 - AP is output per unit of ______ factor.

    एपी ______ कारक के ��त इकाई उ�पादन है।Options:

    1) variable प�रवत�नशील 2) total

    कुल 3) fixed

    ि�थर 4) marginal

    सीमांतCorrect Answer: variable

    प�रवत�नशीलCandidate Answer: variable

    प�रवत�नशील QID : 268 - License fee comes under:

    लाइस�स श�ुक अतंग�त आता हैः

    ww

    w.k

    vcla

    sses

    .com

    Downloaded from www.kvclasses.com

    Visit www.kvclasses.com for tons of free study materials and latest exam updates

  • Options: 1) fixed cost

    ि�थर लागत 2) variable cost

    प�रवत�नीय लागत 3) direct cost

    ��य� लागत 4) prime cost

    मूल लागतCorrect Answer: fixed cost

    ि�थर लागतCandidate Answer: fixed cost

    ि�थर लागत QID : 269 - Rising of AVC is due to:

    एवीसी म� व�� का कारण हैःOptions:

    1) increasing returns to a factor एक कारक के �लए ��तफल बढ़ाना

    2) diminishing returns to a factor कारक के �लए ��तफल घटाना

    3) constant returns to a factor एक कारक के �लए ि�थर ��तफल

    4) no returns to a factor एक कारक के �लए कोई ��तफल नह�ं

    Correct Answer: diminishing returns to a factor कारक के �लए ��तफल घटाना

    Candidate Answer: increasing returns to a factor एक कारक के �लए ��तफल बढ़ाना

    QID : 270 - The demand curve of the firm under perfect competition is:

    पूण� ��तयो�गता के तहत फम� का माँग व� हैःOptions:

    1) greater than one elastic एक लोचदार से अ�धक

    2) unitary elastic एका�मक लोचदार

    3) perfectly elastic पूण�तः लोचदार

    4) perfectly inelastic पूण�तः बेलोच

    Correct Answer: perfectly elastic पूण�तः लोचदार

    Candidate Answer: perfectly inelastic पूण�तः बेलोच

    QID : 271 - A monopolist charges a price which is ______ than the marginal cost.

    एक एका�धकार� एक क�मत वसूलता है जो सीमांत लागत से ______ है।Options:

    1) greater अ�धक

    2) less कम

    3) equal बराबर

    4) None of these इनम� से कोई नह�ं

    Correct Answer: greater अ�धक

    ww

    w.k

    vcla

    sses

    .com

    Downloaded from www.kvclasses.com

    Visit www.kvclasses.com for tons of free study materials and latest exam updates

  • Candidate Answer: greater अ�धक

    QID : 272 - Price control leads to rationing and black marketing under which forms of market?

    �कस�कार के बाज़ार म�, मू�य �नयं�ण राशन तथा काला बाज़ार� को बढ़ावा देता है?Options:

    1) Perfect competition पूण� ��तयो�गता

    2) Oligopoly अ�पा�धकार

    3) Monopoly एका�धकार

    4) Monopolistic competition एका�धकार ��तयो�गता

    Correct Answer: Perfect competition पूण� ��तयो�गता

    Candidate Answer: Monopoly एका�धकार

    QID : 273 - Why does the firms in long run earn normal profits under perfect competition?

    लंबी अव�ध म� कंप�नय� को पूण� ��तयो�गता के अतंग�त सामा�य लाभ �य� �मलता है?Options:

    1) Large number of buyers of a commodity. एक व�तु के �व�ेताओ� क� अ�धक सं�या होना।

    2) Homogenous product is sold by the firms. फम� �वारा एक समान व�तुएँ बेची जाती है।

    3) Perfect knowledge among sellers. �व�ेताओं को पूण� �ान होना।

    4) Freedom of entry and exit of the firms. फम� क� ��वि�ट तथा �नकास क� �वतं�ता होना।

    Correct Answer: Freedom of entry and exit of the firms. फम� क� ��वि�ट तथा �नकास क� �वतं�ता होना।

    Candidate Answer: Large number of buyers of a commodity. एक व�तु के �व�ेताओ� क� अ�धक सं�या होना।

    QID : 274 - What does a perfectly elastic demand curve of a firm reflects under perfect competition?

    पूण� ��तयो�गता के अतंग�त एक फम� का पूण�तः लोचदार माँग व� �या �द�श�त करता है?Options:

    1) Zero price control श�ूय क�मत �नयं�ण

    2) Price maker क�मत देने वाला

    3) Partial market control आं�शक बाज़ार �नयं�ण

    4) Perfect immobility पूण� ग�तह�नता

    Correct Answer: Zero price control श�ूय क�मत �नयं�ण

    Candidate Answer: Zero price control श�ूय क�मत �नयं�ण

    QID : 275 - If oligopoly firms produces differentiated products, then it is a situation of:

    य�द अ�पा�धकार फम� �व�भ�न उ�पाद� का उ�पादन करती ह�, तो यह एक ि�थ�त हैः

    ww

    w.k

    vcla

    sses

    .com

    Downloaded from www.kvclasses.com

    Visit www.kvclasses.com for tons of free study materials and latest exam updates

  • Options: 1) perfect oligopoly

    पूण� अ�पा�धकार 2) imperfect oligopoly

    अपूण� अ�पा�धकार 3) pure monopoly

    पूण� एका�धकार 4) duopoly

    �वय�धकारCorrect Answer: imperfect oligopoly

    अपूण� अ�पा�धकारCandidate Answer: perfect oligopoly

    पूण� अ�पा�धकार QID : 276 - In which form of market, there are only two firms producing a commodity?

    बाजार के �कस �प म�, एक व�तु उ�पा�दत करने वाल� केवल दो ह� फम� ह�?Options:

    1) Monopolistic competition एका�धकार ��तयो�गता

    2) Duopoly �वय�धकार

    3) Monopoly एका�धकार

    4) Oligopoly अ�पा�धकार

    Correct Answer: Duopoly �वय�धकार

    Candidate Answer: Duopoly �वय�धकार

    QID : 277 - Indian Railways can be termed as ______.

    भारतीय रेलवे को ______ भी कहा जा सकता है।Options:

    1) Natural Monopoly �ाकृ�तक एका�धकार

    2) Public Monopoly साव�ज�नक एका�धकार 3) Private Monopoly �नजी एका�धकार

    4) Legal Monopoly कानूनी एका�धकार

    Correct Answer: Public Monopoly साव�ज�नक एका�धकार

    Candidate Answer: Public Monopoly साव�ज�नक एका�धकार

    QID : 278 - Gross National disposable income is equal to:

    सकल रा���य �यो�य आय बराबर हैःOptions:

    1) Net National disposable income + Current replacement cost �नवल (नेट) रा���य �यो�य आय + वत�मान ��त�थापन लागत

    2) Net National disposable income – Corporate profit tax �नवल (नेट) रा���य �यो�य आय – कॉप�रेट लाभ कर

    3) Gross National disposable income – Direct personal taxes सकल रा���य �यो�य आय – ��य� �यि�तगत कर

    4) Gross National disposable income – Miscellaneous fees and fines paid by the households सकल रा���य �यो�य आय – प�रवार� �वारा �द�त �व�वध श�ुक तथा जमुा�ना

    Correct Answer: Net National disposable income + Current replacement cost �नवल (नेट) रा���य �यो�य आय + वत�मान ��त�थापन लागत

    ww

    w.k

    vcla

    sses

    .com

    Downloaded from www.kvclasses.com

    Visit www.kvclasses.com for tons of free study materials and latest exam updates

  • Candidate Answer: Net National disposable income – Corporate profit tax �नवल (नेट) रा���य �यो�य आय – कॉप�रेट लाभ कर

    QID : 279 -

    Options:

    1) I-3, II-4, III-1, IV-2

    2) I-2, II-3, III-4, IV-1

    3) I-4, II-3, III-2, IV-1

    4) I-3, II-1, III-2, IV-4 Correct Answer: I-4, II-3, III-2, IV-1

    Candidate Answer: I-4, II-3, III-2, IV-1

    QID : 280 - The process of moving from self-employment and regular salaried employment to casual wage work is known as ______.

    �वयंरोजगार तथा �नय�मत �प से वेतनभोगी रोजगार से आकि�मक मजदरू� के काय� म� जाने क� ���या को ______ के �प म� जाना जाता है।Options:

    1) casualisation of workforce काय�बल क� आकि�मकता

    2) degeneration of workforce काय�बल के अध:पतन

    3) normalisation of workforce काय�बल का सामा�यीकरण

    4) retrieving of workforce काय�बल को पुन: �ा�त करना

    Correct Answer: casualisation of workforce काय�बल क� आकि�मकता

    Candidate Answer: casualisation of workforce काय�बल क� आकि�मकता

    QID : 281 - GDP deflator equals:

    जीडीपी अप�फ��तकारक बराबर हैः

    ww

    w.k

    vcla

    sses

    .com

    Downloaded from www.kvclasses.com

    Visit www.kvclasses.com for tons of free study materials and latest exam updates

  • Options: 1) [(GDP at current prices)/(GDP at constant prices)] × 100

    [(वत�मान क�मत� पर जीडीपी)/(ि�थर क�मत� पर जीडीपी)] × 100 2) [(GDP at constant prices)/(GDP at current prices)] × 100

    [(ि�थर क�मत� पर जीडीपी)/(वत�मान क�मत� पर जीडीपी)] × 100 3) [(GDP at real prices)/(GDP at constant prices)] × 100

    [(वा�त�वक क�मत� पर जीडीपी)/(ि�थर क�मत� पर जीडीपी)] × 100 4) None of these

    इनम� से कोई नह�ंCorrect Answer: [(GDP at current prices)/(GDP at constant prices)] × 100

    [(वत�मान क�मत� पर जीडीपी)/(ि�थर क�मत� पर जीडीपी)] × 100Candidate Answer: [(GDP at constant prices)/(GDP at current prices)] × 100

    [(ि�थर क�मत� पर जीडीपी)/(वत�मान क�मत� पर जीडीपी)] × 100 QID : 282 - Which industry is also known as basic industry?

    �कस उ�योग को आधारभूत उ�योग भी कहा जाता है?Options:

    1) Chemical Industry रसायन उ�योग

    2) Agro based Industry कृ�ष आधा�रत उ�योग

    3) Forest based Industry वन आधा�रत उ�योग

    4) Iron and Steel Industry लौह तथा इ�पात उ�योग

    Correct Answer: Iron and Steel Industry लौह तथा इ�पात उ�योग

    Candidate Answer: Iron and Steel Industry लौह तथा इ�पात उ�योग

    QID : 283 - In terms of economic system, what is disinvetment?

    अथ�शा�� क� श�दावल� म�, �व�नवेश �या है?Options:

    1) Privatisation of the public sector enterprises साव�ज�नक �े�क के उ�यम� का �नजीकरण

    2) Increasing the value of share of a company कंपनी के अशं� (शयेर�) के मू�य म� व�ृ�

    3) Decreasing the value of share of a company कंपनी के अशं� (शयेर�) के मू�य म� कमी

    4) Foreign investments in defence sector र�ा �े�क म� �वदेशी �नवेश

    Correct Answer: Privatisation of the public sector enterprises साव�ज�नक �े�क के उ�यम� का �नजीकरण

    Candidate Answer: Privatisation of the public sector enterprises साव�ज�नक �े�क के उ�यम� का �नजीकरण

    QID : 284 - When the number of poor is estimated as the proportion of people below the poverty line, it is known as ______.

    जब �नध�न� क� सं�या का अनुमान गर�बी-रेखा से नीचे के लोग� के अनुपात के �प म� अनुमा�नत है, तो इसे ______ कहा जाता है।Options:

    1) Earning Member Ratio सद�य� क� कमाई का अनुपात 2) Head Count Ratio

    �यि�त गणना अनुपात 3) Foot Count Ratio पद गणना अनुपात

    4) Poor People Ratio �नध�न �यि�त अनुपात

    Correct Answer: Head Count Ratio �यि�त गणना अनुपात

    ww

    w.k

    vcla

    sses

    .com

    Downloaded from www.kvclasses.com

    Visit www.kvclasses.com for tons of free study materials and latest exam updates

  • Candidate Answer: Poor People Ratio �नध�न �यि�त अनुपात

    QID : 285 - What is Fiscal deficit?

    राजकोषीय घाटे से �या अ�भ�ाय है?Options:

    1) It occurs when a government's total revenue exceeds the total expenditure excluding borrowings. ऐसा तब होता है जब सरकार क� कुल आय उधार� को छोड़कर कुल �यय से अ�धक हो जाती है।

    2) It is the difference between governments total receipts and its total expenditure including borrowings. यह सरकार क� कुल �ाि�त तथा कुल �यय का अतंर है, उधार रा�शय� को सि�म�लत करके।

    3) It occurs when a government's total expenditure exceed the revenue that it generates, excluding money from borrowings. ऐसा तब होता है जब सरकार का कुल �यय उसके राज�व से अ�धक होता है जो उधार� को छोड़कर उ�प�न होते है।

    4) No option is correct. कोई �वक�प सह� नह�ं है।

    Correct Answer: It occurs when a government's total expenditure exceed the revenue that it generates, excluding money from borrowings. ऐसा तब होता है जब सरकार का कुल �यय उसके राज�व से अ�धक होता है जो उधार� को छोड़कर उ�प�न होते है।

    Candidate Answer: It occurs when a government's total expenditure exceed the revenue that it generates, excluding money from borrowings. ऐसा तब होता है जब सरकार का कुल �यय उसके राज�व से अ�धक होता है जो उधार� को छोड़कर उ�प�न होते है।

    QID : 286 - Which among the following is NOT an example of Heavy industry?

    �न�न�ल�खत म� से कौन सा बड़ ेपैमाने वाले उ�योग का एक उदाहरण नह�ं है?Options:

    1) Metal melting धातु गलाना

    2) Chemical manufacture रसायन उ�पादन

    3) Textile production कपड़ा उ�पादन

    4) Pottery and brick making from clay �म�ी से बत�न तथा �टे बनाना

    Correct Answer: Pottery and brick making from clay �म�ी से बत�न तथा �टे बनाना

    Candidate Answer: Pottery and brick making from clay �म�ी से बत�न तथा �टे बनाना

    QID : 287 - Which among the following is the oldest economic activity known?

    �न�न�ल�खत म� से �ाचीनतम आ�थ�क ��या कौन सी है?Options:

    1) Mining खनन

    2) Cotton weaving कपास क� बुनाई

    3) Agriculture कृ�ष

    4) All of these ये सभी

    Correct Answer: Agriculture कृ�ष

    Candidate Answer: Agriculture कृ�ष

    QID : 288 - Which of the following industry is NOT reserved for the public sector after New Economic Policy of 1991?

    1991 क� नई आ�थ�क नी�त के प�चात �न�न�ल�खत उ�योग म� से कौन सा साव�ज�नक �े� के �लए आर��त नह�ं है?

    ww

    w.k

    vcla

    sses

    .com

    Downloaded from www.kvclasses.com

    Visit www.kvclasses.com for tons of free study materials and latest exam updates

  • Options: 1) Defence equipments

    र�ा उपकरण 2) Railway transport

    रेल प�रवहन 3) Aerospace

    अतं�र� संबंधी 4) Atomic energy generation

    परमाण ुऊजा� उ�पादनCorrect Answer: Aerospace

    अतं�र� संबंधीCandidate Answer: Aerospace

    अतं�र� संबंधी QID : 289 - The trade policy reforms aimed at:

    �यापार नी�त सुधार� का उ�े�य हैःOptions:

    1) dismantling of quantitative restrictions on imports and exports. आयात तथा �नया�त पर मा�ा�मक ��तबंध� को समा�त करना।

    2) reduction of tariff rates. टै�रफ दर म� कमी करना।

    3) removal of licensing procedures for imports. आयात के �लए लाइस��सगं ���याओं को हटाना।

    4) All of these ये सभी

    Correct Answer: All of these ये सभी

    Candidate Answer: All of these ये सभी

    QID : 290 - Under which Amendment Act of Indian Constitution GST is introduced?

    भारतीय सं�वधान के �कस संशोधन अ�ध�नयम के अतंग�त जीएसट� ��तुत �कया गया है?Options:

    1) 100

    2) 101

    3) 102

    4) 105 Correct Answer: 101

    Candidate Answer: 100

    QID : 291 - Who is the chairman of GST Council?

    जीएसट� स�म�त के अ�य� कौन है?Options:

    1) Arun Jaitely अ�ण जेटल�

    2) Pranab Mukherjee �णब मुख़ज�

    3) Narendra Modi नर�� मोद�

    4) Ram Nath Kovind राम नाथ को�वदं

    Correct Answer: Arun Jaitely अ�ण जेटल�

    Candidate Answer: Arun Jaitely अ�ण जेटल�

    QID : 292 - What is the allocation on defence budget in the Union Budget of India (2018-19)?

    भारत के क� ��य बजट (2018-19) म� र�ा बजट पर आवंटन �या है?

    ww

    w.k

    vcla

    sses

    .com

    Downloaded from www.kvclasses.com

    Visit www.kvclasses.com for tons of free study materials and latest exam updates

  • Options: 1) Rs 4.52 lakh crores

    4.52 लाख करोड़ � 2) Rs 2.95 lakh crores

    2.95 लाख करोड़ � 3) Rs 3.68 lakh crores

    3.68 लाख करोड़ � 4) Rs 5.72 lakh crores

    5.72 लाख करोड़ �Correct Answer: Rs 2.95 lakh crores

    2.95 लाख करोड़ �Candidate Answer: Rs 2.95 lakh crores

    2.95 लाख करोड़ � QID : 293 - What is that money which is accepted as a medium of exchange because of the trust between the payer and the payee?

    वह धन �या है जो दाता तथा आदाता के म�य के �व�वास के कारण �व�नमय के मा�यम के �प म� �वीकार �कया जाता है?Options:

    1) Fiat money अ�ध�द�ट धन

    2) Fiduciary money ��ययी धन

    3) Credit money जमा धन

    4) Full bodied money पूण� मू�य धन

    Correct Answer: Fiduciary money ��ययी धन

    Candidate Answer: Fiduciary money ��ययी धन

    QID : 294 - Currency deposit ratio equals:

    मु�ा जमा अनुपात बराबर हैःOptions:

    1) Cash in hand with the people लोग� के साथ रोकड़ शषे

    2) Demand deposit with the people लोग� के पास माँग जमा रा�श

    3) Both (A) and (B) (A) तथा (B) दोन�

    4) None of these इनम� से कोई नह�ं

    Correct Answer: Both (A) and (B) (A) तथा (B) दोन�

    Candidate Answer: [ NOT ANSWERED ] QID : 295 - Credit multiplier is equal to:

    ऋण गुणक बराबर हैःOptions:

    1) 1/CRR 1/सीआरआर

    2) 1/SLR 1/एसएलआर

    3) 1/Repo Rate 1/रेपो दर

    4) 1/Reverse Repo Rate 1/�रवस� रेपो दर

    Correct Answer: 1/CRR 1/सीआरआर

    Candidate Answer: 1/CRR 1/सीआरआर

    ww

    w.k

    vcla

    sses

    .com

    Downloaded from www.kvclasses.com

    Visit www.kvclasses.com for tons of free study materials and latest exam updates

  • QID : 296 - In 1991, as an immediate measure to resolve the balance of payments crisis, the rupee was ______ against foreign currencies.

    1991 म�, भुगतान संकट के संतुलन को हल करने के �लए त�काल उपाय के �प म�, �पया �वदेशी मु�ाओं के �व�� ______ था।Options:

    1) revalued पुनमू��यांकन 2) devalued अवमू�यन

    3) freezed ि�थर

    4) freed मु�त

    Correct Answer: devalued अवमू�यन

    Candidate Answer: devalued अवमू�यन

    QID : 297 -

    Options:

    1) I-1, II-3, III-2

    2) I-2, II-1, III-3

    3) I-3, II-1, III-2

    4) I-3, II-2, III-1 Correct Answer: I-3, II-1, III-2

    Candidate Answer: I-2, II-1, III-3

    QID : 298 - Prime Minister of India has launched an online platform mygov.nic.in to engage citizens in the task of _____.

    भारत के �धानमं�ी ने नाग�रक� को _____ काय� म� संल�न करने के �लए एक ऑन-लाइन मंच mygov.nic.in श�ु �कया है।

    ww

    w.k

    vcla

    sses

    .com

    Downloaded from www.kvclasses.com

    Visit www.kvclasses.com for tons of free study materials and latest exam updates

  • Options: 1) Sampoorn

    संपूण� 2) Sunehra

    सुनहेरा 3) Suraksha

    सुर�ा 4) Surajya

    सुरा�यCorrect Answer: Surajya

    सुरा�यCandidate Answer: Surajya

    सुरा�य QID : 299 - Which organization has launched the BHIM mobile application?

    �कस संगठन ने भीम मोबाइल एि�लकेशन लोका�प�त क� है?Options:

    1) NPCI एनपीसीआई

    2) NITI Ayog नी�त आयोग

    3) NSDL एनएसडीएल

    4) NIC एनआईसी

    Correct Answer: NPCI एनपीसीआई

    Candidate Answer: NIC एनआईसी

    QID : 300 - Who will be appointed as the president of NASSCOM, upon the completion of R. Chandrashekhar's term in March, 2018?

    माच� 2018 म� आर. च�ंशखेर के काय�काल के प


Recommended